Question and Answers Forum

All Questions      Topic List

Others Questions

Previous in All Question      Next in All Question      

Previous in Others      Next in Others      

Question Number 20842 by Tinkutara last updated on 04/Sep/17

Acceleration of a particle which is at  rest at x = 0 is a^→  = (4 − 2x) i^∧ . Select  the correct alternative(s).  (a) Maximum speed of the particle is  4 units  (b) Particle further comes to rest at  x = 4  (c) Particle oscillates about x = 2  (d) Particle will continuously accelerate  along the x-axis.

$$\mathrm{Acceleration}\:\mathrm{of}\:\mathrm{a}\:\mathrm{particle}\:\mathrm{which}\:\mathrm{is}\:\mathrm{at} \\ $$$$\mathrm{rest}\:\mathrm{at}\:{x}\:=\:\mathrm{0}\:\mathrm{is}\:\overset{\rightarrow} {{a}}\:=\:\left(\mathrm{4}\:−\:\mathrm{2}{x}\right)\:\overset{\wedge} {{i}}.\:\mathrm{Select} \\ $$$$\mathrm{the}\:\mathrm{correct}\:\mathrm{alternative}\left(\mathrm{s}\right). \\ $$$$\left({a}\right)\:\mathrm{Maximum}\:\mathrm{speed}\:\mathrm{of}\:\mathrm{the}\:\mathrm{particle}\:\mathrm{is} \\ $$$$\mathrm{4}\:\mathrm{units} \\ $$$$\left({b}\right)\:\mathrm{Particle}\:\mathrm{further}\:\mathrm{comes}\:\mathrm{to}\:\mathrm{rest}\:\mathrm{at} \\ $$$${x}\:=\:\mathrm{4} \\ $$$$\left({c}\right)\:\mathrm{Particle}\:\mathrm{oscillates}\:\mathrm{about}\:{x}\:=\:\mathrm{2} \\ $$$$\left({d}\right)\:\mathrm{Particle}\:\mathrm{will}\:\mathrm{continuously}\:\mathrm{accelerate} \\ $$$$\mathrm{along}\:\mathrm{the}\:{x}-\mathrm{axis}. \\ $$

Answered by ajfour last updated on 09/Sep/17

((v_x dv_x )/dx)=4−2x  ⇒ ∫_0 ^(  v_x ) v_x dv_x =∫_0 ^(  x) (4−2x)dx  ⇒   (v_x ^2 /2)=4x−x^2   ⇒    v_x ^2 =2[4−(x−2)^2 ]  ⇒ particle comes to rest at x=4.  maximum speed at x=2 and is  (v_x )_(max) =2(√2) units.  ⇒  ∫_0 ^(  x)  (dx/(√(2^2 −(x−2)^2 ))) = ±(√2)∫_0 ^( t) dt  ⇒sin^(−1) (((x−2)/2))−sin^(−1) (−1)=±(√2)t  ⇒  sin^(−1) (((x−2)/2))=±(√2)t−π/2  x=2+2sin (±(√2)t−π/2)  or    x=2−cos ((√2)t)     is the equation of motion.  ⇒  particle oscillates about x=2.  (b) and (c) are the correct options.

$$\frac{{v}_{{x}} {dv}_{{x}} }{{dx}}=\mathrm{4}−\mathrm{2}{x} \\ $$$$\Rightarrow\:\int_{\mathrm{0}} ^{\:\:{v}_{{x}} } {v}_{{x}} {dv}_{{x}} =\int_{\mathrm{0}} ^{\:\:{x}} \left(\mathrm{4}−\mathrm{2}{x}\right){dx} \\ $$$$\Rightarrow\:\:\:\frac{{v}_{{x}} ^{\mathrm{2}} }{\mathrm{2}}=\mathrm{4}{x}−{x}^{\mathrm{2}} \\ $$$$\Rightarrow\:\:\:\:{v}_{{x}} ^{\mathrm{2}} =\mathrm{2}\left[\mathrm{4}−\left({x}−\mathrm{2}\right)^{\mathrm{2}} \right] \\ $$$$\Rightarrow\:{particle}\:{comes}\:{to}\:{rest}\:{at}\:{x}=\mathrm{4}. \\ $$$${maximum}\:{speed}\:{at}\:{x}=\mathrm{2}\:{and}\:{is} \\ $$$$\left({v}_{{x}} \right)_{{max}} =\mathrm{2}\sqrt{\mathrm{2}}\:{units}. \\ $$$$\Rightarrow\:\:\int_{\mathrm{0}} ^{\:\:{x}} \:\frac{{dx}}{\sqrt{\mathrm{2}^{\mathrm{2}} −\left({x}−\mathrm{2}\right)^{\mathrm{2}} }}\:=\:\pm\sqrt{\mathrm{2}}\int_{\mathrm{0}} ^{\:{t}} {dt} \\ $$$$\Rightarrow\mathrm{sin}^{−\mathrm{1}} \left(\frac{{x}−\mathrm{2}}{\mathrm{2}}\right)−\mathrm{sin}^{−\mathrm{1}} \left(−\mathrm{1}\right)=\pm\sqrt{\mathrm{2}}{t} \\ $$$$\Rightarrow\:\:\mathrm{sin}^{−\mathrm{1}} \left(\frac{{x}−\mathrm{2}}{\mathrm{2}}\right)=\pm\sqrt{\mathrm{2}}{t}−\pi/\mathrm{2} \\ $$$${x}=\mathrm{2}+\mathrm{2sin}\:\left(\pm\sqrt{\mathrm{2}}{t}−\pi/\mathrm{2}\right) \\ $$$${or}\:\:\:\:{x}=\mathrm{2}−\mathrm{cos}\:\left(\sqrt{\mathrm{2}}{t}\right) \\ $$$$\:\:\:{is}\:{the}\:{equation}\:{of}\:{motion}. \\ $$$$\Rightarrow\:\:{particle}\:{oscillates}\:{about}\:{x}=\mathrm{2}. \\ $$$$\left(\boldsymbol{{b}}\right)\:{and}\:\left(\boldsymbol{{c}}\right)\:{are}\:{the}\:{correct}\:{options}. \\ $$

Commented by ajfour last updated on 09/Sep/17

thanks, i shall be careful.

$${thanks},\:{i}\:{shall}\:{be}\:{careful}. \\ $$

Commented by alex041103 last updated on 09/Sep/17

You have a mistake  ⇒  ∫_0 ^(  x)  (dx/(√(2^2 −(x−2)^2 ))) = ±(√2)∫_0 ^( t) dt    ⇒x(t)=2−2cos((√2)t)

$${You}\:{have}\:{a}\:{mistake} \\ $$$$\Rightarrow\:\:\int_{\mathrm{0}} ^{\:\:{x}} \:\frac{{dx}}{\sqrt{\mathrm{2}^{\mathrm{2}} −\left({x}−\mathrm{2}\right)^{\mathrm{2}} }}\:=\:\pm\sqrt{\mathrm{2}}\int_{\mathrm{0}} ^{\:{t}} {dt} \\ $$$$ \\ $$$$\Rightarrow{x}\left({t}\right)=\mathrm{2}−\mathrm{2}{cos}\left(\sqrt{\mathrm{2}}{t}\right) \\ $$

Commented by Tinkutara last updated on 04/Sep/17

Thank you very much Sir!

$$\mathrm{Thank}\:\mathrm{you}\:\mathrm{very}\:\mathrm{much}\:\mathrm{Sir}! \\ $$

Terms of Service

Privacy Policy

Contact: info@tinkutara.com